Está en la página 1de 4

MATH 267 Homework Assignment 5 Solutions

1. Consider the RL circuit


x(t)

y(t)
1H

A current source produces an input current x(t) and the system output is considered to be the current,
y(t), flowing through the inductor.
(a) Find the differential equation relating y(t) to x(t).
1
(b) Assume that x(t) is periodic with period 60
sec. Find a formula relating the Fourier coefficients of
y(t) to the Fourier coefficients of x(t).
(c) What implicit assumptions have you made in arriving at your answer to part (b)?
Solution. (a) By conservation of current, the current flowing through the resistor is x y. Since the
voltage across the resistor must be the same as the voltage across the inductor
xy =

dy
dt

= y 0 + y = x

(b) Since the input signal is periodic with 60 cycles per second
x(t) =

ck ei120kt

dk ei120kt

k=

Then
y(t) =

k=

is a solution if and only if


y 0 (t) + y(t) =

(120ki + 1)dk ei120kt = x(t) =

k=

ck ei120kt

k=

which is the case if and only if


dk =

ck
1+120ki

(c) The general solution to y 0 + y = x is the sum of a particular solution, which we found in part (b),
plus the general solution to the homogeneous equation, which is Cet . We have implicitly assumed that
C is forced to be zero, or that we are not interested in this transient part of the solution because we
are interested in times large enough that it has become small enough to be ignored.
2. The output voltage y(t) of an RLC circuit connected to an external voltage source satisfies the equation
LCy 00 (t) + RCy 0 (t) + y(t) = x(t). Suppose that x(t) is the triangular wave of period 2 given by x(t) = |t|
for 1 t 1. Find y(t).
Optional: Plot the shape of the output voltage y(t) for 4 t 4
(a) for R = 1, L = 1 and C = 0,
(b) for R = 1, L = 1 and C = 0.01 and
(c) for R = 1, L = 1 and C = 0.02.
1

using the first 10 non-zero terms in the Fourier series for y(t).
Solution. First we find (the complex exponential form of) the Fourier series for
x(t) =

ck eikt

with

ck =

1
2

k=

|t|eikt dt =

1
2

(t)eikt dt +

1
2

teikt dt
0

For k = 0, since |t| is an even function,


1
2

c0 = 2

t dt =
0

1
2

Now consider k 6= 0. Then


ck = 21

ikt
t
ik e

ikt
1
(ik)2 e

i0

1
2

ikt
t
ik e

ikt
1
(ik)2 e

i1
0

For k even, eikt = 1 for all of t = 0, 1, 1 so that


ck = 21

t
ik

1
(ik)2

i0

1
2

t
ik

1
(ik)2

i1
0

1 1
2 ik


1+1 =0

For k odd, eikt is 1 for t = 0 and 1 for t = 1, 1 so that


ck =

1 1
2 ik


0+11+0 +

1
1
2 (ik)2


1 + 1 + 1 + 1 = k222

All together

1
if k = 0

2
ck = 0
if k 6= 0 is even,

2
2 k2 if k is odd.
P
Call y(t)s coefficients dk so that y(t) = k= dk eikt . The ODE LCy 00 (t) + RCy 0 (t) + y(t) = x(t) is
satisfied if and only if

k=

X

LC 2 k 2 + iRCk + 1 dk eikt =
ck eikt
k=

Matching coefficients gives

so that


LC 2 k 2 + iRCk + 1 dk = ck

2
ck
dk =
= 0
2
LC 2 k 2 + iRCk + 1

(LC 2 k 2 + iRCk + 1) 2 k 2

if k = 0
if k 6= 0 and k is even
if k is odd

Notice that C = 0 just reproduces the function f (t), so the first plot should be a triangle wave. Here
are the three plots.

RLC circuit output with C = 0


1.5
1
0.5
0
0.5
4

RLC circuit output with C = 0.01


1.5
1
0.5
0
0.5
4

RLC circuit output with C = 0.02


1.5
1
0.5
0
0.5
4

3. Suppose that L = R = 1 in the previous problem. What value of C should you choose to tune in
the frequency ei5t ? I.e. what value of C gives the largest possible amplitude of the response y(t), if
x(t) = ei5t ?
Solution. The amplitude of the transfer function for frequencey = k is given by




1
1


LC 2 + iRC + 1 = q
2
R2 C 2 2 + (1 LC 2 )
This attains is maximum when
2

R2 C 2 2 + (1 LC 2 )2 attains its minimum, or equivalently when

R2 C 2 2 + (1 LC 2 ) attains its minimum. This is the case when


0=

d
d

R2 C 2 2 + (1 LC 2 )

2

= 2R2 C 2 4LC(1 LC 2 ) = 1 LC 2 =

Substituting in = 5 and L = R = 1 gives


1 25 2 C = 12 C = C =

2
50 2 +1

4. Consider the non-homogeneous differential equation


y 00 (t) + y 0 (t) + y(t) = x(t)

R2 C
2L

or = 0

where x(t) is the discontinuous square wave function of period 2 given by



1 if 1 t < 0
x(t) =
1
if 0 t < 1
Let y(t) be the solution obtained by first finding the Fourier series for x(t) in complex exponential form.
(a) Write down the series for y(t) in complex exponential form.
(b) Write down the real form of the series obtained by taking the real part of each term.
P
Solution. (a) The Fourier expansion of x(t) is x(t) = k= ck eikt with
Z 1
Z 1
Z 0
x(t)eikt dt = 12
eikt dt + 12
(1)eikt dt
ck = 12
0

For k = 0
ck =

1
2

dt
0

1
2

dt =

1
2

1
2

=0

For k 6= 0
ck =

 1 ikt 1
1
2 ik e
0

All together,

 1 ikt 0
1
2 ik e
1

i
2k

 ik

e
1 1 + eik(1) =

i
k

(1)k 1

if k = 0

0
if k 6= 0 is even,
ck = 0

2i
k if k is odd.
P
Call y(t)s coefficients dn so that y(t) = k= dk eikt . The ODE y 00 (t) + y 0 (t) + y(t) = x(t) is
satisfied if and only if

k=

X

2 k 2 + ik + dk eikt =
ck eikt
k=

Matching coefficients gives



2 k 2 + ik + dk = ck

so that
y(t) =

k=
k odd

2i
eikt
k( k 2 2 + ik)

(b) Taking real parts:


Re y(t) = Re

k=
k odd

k[(

2
k2 2 )

+ (k)2 ]

i cos(kt) + sin(kt)

X
2( k 2 2 ) sin(kt) 2k cos(kt)
2

k=
k odd

2( k 2 2 ik)

k[( k 2 2 ) + 2 k 2 2 ]

X
4( (2n 1)2 2 ) sin((2n 1)t) 2(2n 1) cos((2n 1)t)

n=1

(2n 1)[( (2n 1)2 2 )2 + 2 (2n 1)2 2 ]

For the last equality, we used that if Bk = Bk , then

k=
k odd

Bk = 2

k=1
k odd

Bk = 2

n=1

B2n1

También podría gustarte